First order linear differential equation

Click For Summary
SUMMARY

The forum discussion centers on solving the first order linear differential equation dy/dt = k*y*ln(y/M), where M and k are constants. The participant successfully demonstrates that the solution y = Meaekt satisfies the equation by manipulating the logarithmic and exponential forms. Key steps included taking the derivative and substituting back into the original equation, confirming the solution is valid. The discussion highlights the importance of understanding logarithmic identities in solving differential equations.

PREREQUISITES
  • Understanding of first order linear differential equations
  • Familiarity with logarithmic and exponential functions
  • Knowledge of differentiation techniques
  • Basic concepts of constants in mathematical equations
NEXT STEPS
  • Study the method of integrating factors for first order differential equations
  • Explore the applications of differential equations in real-world scenarios
  • Learn about the stability of solutions in differential equations
  • Investigate advanced techniques for solving nonlinear differential equations
USEFUL FOR

Students and educators in mathematics, particularly those focusing on differential equations, as well as anyone interested in applying mathematical concepts to solve real-world problems.

physicsernaw
Messages
41
Reaction score
0

Homework Statement



dy/dt = k*y*ln(y/M), where M and k are constants.

Show that y = Meaekt satisfies the above equation for any constant a.

Homework Equations



y' = ky
y = P0ekt

The Attempt at a Solution



Taking the derivative of y, I get:

(Meaekt)*(aekt)*k

which is,

ky*aekt

..and I'm stuck here.

EDIT:

Nevermind, I figured it out.

ln(y) = ln(M) + ae^(kt)

ae^(kt) = ln(y/M)

Plugging back in...

ky*ln(y/M)

:)
 
Last edited:
Physics news on Phys.org
Well done! :)
 
Question: A clock's minute hand has length 4 and its hour hand has length 3. What is the distance between the tips at the moment when it is increasing most rapidly?(Putnam Exam Question) Answer: Making assumption that both the hands moves at constant angular velocities, the answer is ## \sqrt{7} .## But don't you think this assumption is somewhat doubtful and wrong?

Similar threads

  • · Replies 7 ·
Replies
7
Views
2K
Replies
2
Views
2K
  • · Replies 2 ·
Replies
2
Views
2K
  • · Replies 8 ·
Replies
8
Views
2K
  • · Replies 5 ·
Replies
5
Views
1K
Replies
14
Views
2K
  • · Replies 12 ·
Replies
12
Views
3K
Replies
2
Views
1K
Replies
3
Views
2K